2025 AMC 12A Problems/Problem 18: Difference between revisions
Lanouzhihun (talk | contribs) |
Krithikrokcs (talk | contribs) No edit summary |
||
| Line 43: | Line 43: | ||
~ScoutViolet | ~ScoutViolet | ||
==Solution 4 (Bounding)== | |||
Note that if <imath>x,y,z>2,</imath> then this must hold, as if it didn't, we would need, WLOG, <imath>x>3y,</imath> which would imply <imath>x>8.</imath> Therefore, there are at least <imath>6\times 5\times 4=120</imath> solutions. However, we must have <imath>x,y,z\ge2,</imath> as if any variable is <imath>0,</imath> this clearly doesn't work, and <imath>x=1</imath> gives <imath>y>z</imath> and <imath>z>y,</imath> which is impossible. Therefore, there are at most <imath>7\times6\times5=210</imath> solutions. The only choice that works from here is <imath>\boxed{\textbf{(C)}~186}</imath> | |||
==Video Solution 1 by OmegaLearn== | ==Video Solution 1 by OmegaLearn== | ||
https://youtu.be/OPQbAyYZBtA | https://youtu.be/OPQbAyYZBtA | ||
Revision as of 10:41, 7 November 2025
Problem 19
How many ordered triples
of different positive integers less than or equal to
satisfy
,
, and
?
Solution 1
let 0<=x<y<z<=8; x cannot be 0 because it makes xy>z --> 0>z; x cannot be 1 because it makes xy>z ---> y>z;
x=2, y=3, z can be 4, 5 but not others; x=2, y=4, z can be 5, 6, 7; x=2, y=5, z can be 6, 7, 8; x=2, y=6, z can be 7, 8; x=2, y=7, z can be 8; for x=2, total 11 cases;
similarly, for x=3,y=4, 5, 6, 7, total 10 cases; for x=4, y =5, 6, 7, total 6 cases; x=5, y=6, 7, 3 cases; x=6, y=7, z=8, 1 cases;
total = 11 + 10 +6 +3 +1 = 31. permutate x, y, z for ordered triple, it is 31*6=186, C.
~imagination
Solution 2
For now, assume
.
First note that no number can be 0, as it would imply
. Similarly, no number can be 1, as it would imply
. So we only need to consider numbers between 2 and 8, inclusive.
We may use complementary counting:
Consider when
. This implies
. Some quick calculations gives us the products
. We may now calculate the number of times each happens (we are no longer assuming
):
: This case is invalid as it asks for distinct integers.
:
. Then we have
cases.
:
. Then we have
cases.
In total, there are
total cases, so our final answer is
.
~SilverRush
Solution 3
Note that if
, all pairs work - hence, we have
pairs.
Now, note that if
, we get
and
, contradiction. Therefore, assume
(since we said
). Note that we need
. WLOG assume
, we get there are
pairs that work (we just need
):
. With
, we can re-arrange these in
ways each, hence the answer is just
.
~ScoutViolet
Solution 4 (Bounding)
Note that if
then this must hold, as if it didn't, we would need, WLOG,
which would imply
Therefore, there are at least
solutions. However, we must have
as if any variable is
this clearly doesn't work, and
gives
and
which is impossible. Therefore, there are at most
solutions. The only choice that works from here is